Inscription / Connexion Nouveau Sujet
Niveau maths spé
Partager :

Où est l'erreur ?

Posté par
infophile
16-10-09 à 16:23

Bonjour,

J'ai le résultat suivant à démontrer, j'ai trouvé 2 démonstrations mais l'une aboutit à une contradiction...

Soit \chi\in K[X], montrer l'existence et l'unicité de P\in K[X] tel que P|\chi et \chi|P^r.


Démo n°1 :

\chi est le polynôme caractéristique de sa matrice compagnon A, soit P le polynôme minimal de A, on sait que P|\chi (par Cayley-Hamilton) et \chi|P^n.

Démo n°2 :

\chi se décompose en produit de polynôme irréductibles \chi=\chi_1^{m_1}\chi_2^{m_2}\cdots \chi_s^{m_s}, P=\chi_1\chi_2\cdots \chi_s convient (on prend pour r le ppcm des ordre de multiplicité).

Dans la suite on prend pour \chi le polynôme caractéristique de A donnée, et P dont on vient de montrer l'existence. Et on pose B=P(A) puis on raisonne modulo B. Mon problème est que dans ma démo n°1 P(A)=0...

Merci

Posté par
Camélia Correcteur
re : Où est l'erreur ? 16-10-09 à 16:35

Bonjour

Il y a un problème dans la définition de r... L'énoncé devrait être montrer qu'il existe r minimal? unique? et P unique tel que... Spontanément, j'aurais pris la démo 2, dans 1) tu as pris r=n... Mais même dans 2) s'il n'y a que des racines simples, tu as P(A)=0. Est-ce un inconvénient? Enfin, tu as mis un corps K quelconque donc il n'y a pas de raison que ça se décompose!

En fait, plus je regarde et moins je comprends ce que l'on cherche...

Posté par
infophile
re : Où est l'erreur ? 16-10-09 à 16:41

Bonjour Camélia

Quelques précisions de l'énoncé :

- K est un sous-corps de \mathbb{C}
- Tout polynôme de K[X] s'écrit de manière unique comme produit de polynômes irréductibles de K[X]
- A la place de \chi|P^r il est écrit qu'il divise une puissance de P.

Posté par
Camélia Correcteur
re : Où est l'erreur ? 16-10-09 à 16:52

Ah, bon... donc a priori il n'y a pas de matrices!

Toujours est-il que je ne comprends pas l'unicité!

Si je prends Q=(X-1)^3(X-2)^2, Q lui-même convient, mais aussi (X-1)(X-2), (X-1)^2(X-2) et une floppée d'autres!

Si Q=Q_1^{m_1}...Q_s^{m_s} et si P=Q_1^{n_1}...Q_s^{n_s} avec 1\leq n_k\leq m_k on a bien P|Q et Q|P^{sup(m_k)}

Posté par
infophile
re : Où est l'erreur ? 16-10-09 à 17:01

Oui voilà c'est l'unicité qui me gêne, erreur d'énoncé alors ?

Merci Camélia

Posté par
Camélia Correcteur
re : Où est l'erreur ? 17-10-09 à 14:01

Tel quel, oui... Mais j'ai l'impression que tu es au milieu d'une autre histoire, et si tes polynômes ne sont pas quelconques...



Vous devez être membre accéder à ce service...

Pas encore inscrit ?

1 compte par personne, multi-compte interdit !

Ou identifiez-vous :


Rester sur la page

Inscription gratuite

Fiches en rapport

parmi 1675 fiches de maths

Désolé, votre version d'Internet Explorer est plus que périmée ! Merci de le mettre à jour ou de télécharger Firefox ou Google Chrome pour utiliser le site. Votre ordinateur vous remerciera !